Απρόσμενη καθετότητα

Συντονιστές: vittasko, silouan, rek2

Κω.Κωνσταντινίδης
Δημοσιεύσεις: 42
Εγγραφή: Πέμ Μαρ 22, 2018 5:40 pm

Απρόσμενη καθετότητα

#1

Μη αναγνωσμένη δημοσίευση από Κω.Κωνσταντινίδης » Κυρ Οκτ 27, 2019 3:43 pm

Δίνεται τρίγωνο ABC με ορθόκεντρο H και περίκεντρο O.Έστω D,E,F τα ίχνη των υψών του τριγώνου στις BC,CA,AB αντίστοιχα. Η παράλληλη από το H στην BC τέμνει την EF στο X. Να αποδειχθεί ότι \angle XDO=90.
τελευταία επεξεργασία από Κω.Κωνσταντινίδης σε Κυρ Οκτ 27, 2019 4:56 pm, έχει επεξεργασθεί 1 φορά συνολικά.


Κωνσταντινίδης Κωνσταντίνος

Λέξεις Κλειδιά:
Λάμπρος Κατσάπας
Δημοσιεύσεις: 838
Εγγραφή: Σάβ Ιουν 17, 2017 10:17 pm
Τοποθεσία: Αθήνα

Re: Απρόσμενη καθετότητα

#2

Μη αναγνωσμένη δημοσίευση από Λάμπρος Κατσάπας » Κυρ Οκτ 27, 2019 4:54 pm

Κω.Κωνσταντινίδης έγραψε:
Κυρ Οκτ 27, 2019 3:43 pm
Δίνεται τρίγωνο ABC με ορθόκεντρο H. Έστω D,E,F τα ίχνη των υψών του τριγώνου στις BC,CA,AB αντίστοιχα. Η παράλληλη από το H στην BC τέμνει την EF στο X. Να αποδειχθεί ότι \angle XDO=90.
O;


min##
Δημοσιεύσεις: 342
Εγγραφή: Τρί Απρ 18, 2017 3:40 pm

Re: Απρόσμενη καθετότητα

#3

Μη αναγνωσμένη δημοσίευση από min## » Κυρ Οκτ 27, 2019 4:56 pm

(Μια straightforward λύση):
Ας είναι Y η τομή BC,EF.
Από αρμονικές,η AX διχοτομεί την DY.
Πάλι από αρμονικές,αν Z η τομή της EF με την εκ του A παράλληλη στην BC θα είναι XDA\angle=ADZ\angle.(D(X,Z,A,Y)=-1).
Έτσι αρκεί ZDA,ODM όμοια.
Αρκεί δηλαδή   \dfrac{AZ} {AD}=\dfrac{OM} {MD}  δηλαδή αν Z' η τομή της εφαπτομένης του (ABC) στο A με την BC αρκεί
\dfrac {AZ} {AZ'}\dfrac {AZ'} {AD}=\dfrac {OM} {MD} <=>\dfrac {OM} {OC}\dfrac {1} {sin(B-C)}=\dfrac {OM} {MD}.
Τελικά από 2ο Θ.Διαμέσων καταλήγει σε sin(b-c)=\dfrac {b^2-c^2} {2aR} το οποίο είναι απλό


Άβαταρ μέλους
ΦΩΤΙΑΔΗΣ ΠΡΟΔΡΟΜΟΣ
Δημοσιεύσεις: 921
Εγγραφή: Πέμ Νοέμ 22, 2018 9:43 pm

Re: Απρόσμενη καθετότητα

#4

Μη αναγνωσμένη δημοσίευση από ΦΩΤΙΑΔΗΣ ΠΡΟΔΡΟΜΟΣ » Δευ Δεκ 09, 2019 8:02 pm

Κω.Κωνσταντινίδης έγραψε:
Κυρ Οκτ 27, 2019 3:43 pm
Δίνεται τρίγωνο ABC με ορθόκεντρο H και περίκεντρο O.Έστω D,E,F τα ίχνη των υψών του τριγώνου στις BC,CA,AB αντίστοιχα. Η παράλληλη από το H στην BC τέμνει την EF στο X. Να αποδειχθεί ότι \angle XDO=90.
Καλησπέρα Κωνσταντίνε!

Μια με περισσότερες πράξεις:

Έστω M το μέσο του BC και T\equiv EF\cap BC.Επειδή \angle ADC=90^{\circ} αρκεί  \angle HDX=\angle ODM ή \dfrac{XH}{DH}=\dfrac{OM}{DM}.\Delta FHX\sim \Delta FTC\Rightarrow \dfrac{XH}{TC}=\dfrac{FH}{FC}\Leftrightarrow XH=\dfrac{TC\cdot FH}{FC}
Επειδή (T,D/B,C)=-1 (αρμονική τετράδα) είναι \dfrac{TC}{TC-a}=\dfrac{DC}{BD}\Leftrightarrow TC=\dfrac{aDC}{DC-BD}=\dfrac{ab\cos C}{b\cos C-c\cos B}=..=\dfrac{a\left ( b^2+a^2-c^2 \right )}{2\left ( b^2-c^2 \right )}
Επίσης \Delta AFH\sim \Delta ABD\Rightarrow \dfrac{FH}{BD}=\dfrac{AF}{AD}\Leftrightarrow \dfrac{FH}{FC}=\dfrac{BD\cdot AF}{FC\cdot AD}=\cot A\cot B
Άρα τελικά αρκεί \cot A \cot B \cdot \dfrac{a\left ( b^2+a^2-c^2 \right )}{2\left ( b^2-c^2 \right )}\cdot \dfrac{1}{DH}=\dfrac{\cot A \cdot \dfrac{a}{2}}{\dfrac{a}{2}-\dfrac{a^2+c^2-b^2}{2a}}\Leftrightarrow \cot B\cdot \dfrac{1}{\cot C \cdot BD}=\dfrac{2a}{b^2+a^2-c^2}\Leftrightarrow .. \Leftrightarrow \dfrac{\dfrac{\cos B}{\sin B}}{\dfrac{\cos C}{\sin C}}=\dfrac{2a}{b^2+a^2-c^2}\cdot c\cdot \cos B\Leftrightarrow \dfrac{\cos B}{\cos C}\cdot \dfrac{c}{b}=\dfrac{2a}{2ab\cos C}\cdot c \cos B\Leftrightarrow \dfrac{c}{b}=\dfrac{c}{b}
που ισχύει άρα πράγματι \angle XDO=90^{\circ}

183.PNG
183.PNG (27.35 KiB) Προβλήθηκε 1204 φορές
.


Άβαταρ μέλους
ΦΩΤΙΑΔΗΣ ΠΡΟΔΡΟΜΟΣ
Δημοσιεύσεις: 921
Εγγραφή: Πέμ Νοέμ 22, 2018 9:43 pm

Re: Απρόσμενη καθετότητα

#5

Μη αναγνωσμένη δημοσίευση από ΦΩΤΙΑΔΗΣ ΠΡΟΔΡΟΜΟΣ » Τρί Δεκ 10, 2019 6:14 pm

Γενίκευση:

Έστω τρίγωνο ABC και AD ύψος.Έστω P σημείο του ύψους και E\equiv BP\cap AC,F\equiv CP\cap AB.Αν η παράλληλη από το P στην BC τέμνει την EF στο X και Y το ισογώνιο συζηγές σημείο του X ως προς το τρίγωνο ABC να δειχθεί ότι \angle XDY =90^{\circ}

Δεν έχω λύση προς το παρόν :roll: .


Μπάμπης Στεργίου
Επιμελητής
Δημοσιεύσεις: 5561
Εγγραφή: Δευ Δεκ 22, 2008 2:16 pm
Τοποθεσία: Χαλκίδα - Καρδίτσα

Re: Απρόσμενη καθετότητα

#6

Μη αναγνωσμένη δημοσίευση από Μπάμπης Στεργίου » Τρί Δεκ 10, 2019 8:38 pm

Καλησπέρα !

Αν κάποιος βρει λίγο χρόνο, ας δει το εξής :

Πώς προκύπτει η καθετότητα OT\perp XT στο 3ο μήνυμα στο θέμα :

https://artofproblemsolving.com/communi ... 5p12457430


Άβαταρ μέλους
vittasko
Επιμελητής
Δημοσιεύσεις: 2230
Εγγραφή: Πέμ Ιαν 08, 2009 8:46 am
Τοποθεσία: Μαρούσι - Αθήνα.
Επικοινωνία:

Re: Απρόσμενη καθετότητα

#7

Μη αναγνωσμένη δημοσίευση από vittasko » Τρί Δεκ 10, 2019 11:09 pm

Μπάμπης Στεργίου έγραψε:
Τρί Δεκ 10, 2019 8:38 pm
Καλησπέρα !

Αν κάποιος βρει λίγο χρόνο, ας δει το εξής :

Πώς προκύπτει η καθετότητα OT\perp XT στο 3ο μήνυμα στο θέμα :

https://artofproblemsolving.com/communi ... 5p12457430
Μπάμπη και λοιποί φίλοι, καλησπέρα.

Ίσως κάτι να μου ξεφεύγει, αλλά νομίζω ότι λείπουν αρκετά βήματα τεκμηρίωσης.

\bullet Έστω (T) ο περίκυκλος του εγγραψίμου τετραπλεύρου AEHF με διάμετρο AH και έστω το σημείο Z\equiv (O)\cap (T) όπου (O) είναι ο περίκυκλος του δοσμένου τριγώνου \vartriangle ABC.

Από AZ\perp ZH έχουμε ότι η ευθεία ZH περνάει από το αντιδιαμετρικό σημείο A' του A στον κύκλο (O).

Από το A' περνάει και η ευθεία HM, όπου M είναι το μέσον της πλευράς BC, από AT = TH και AO = OA' και HM\parallel TO, λόγω του παραλληλογράμμου TOMH, από TH\parallel = OM ( γνωστό αποτέλεσμα ).

Άρα, τα σημεία Z,\ H, M είναι συνευθειακά και έστω το σημείο S\equiv EF\cap ZH.
f 181_t 65483.PNG
Απρόσμενη καθετότητα - Απόδειξη καθετότητας τωνXT, TO.
f 181_t 65483.PNG (32.71 KiB) Προβλήθηκε 892 φορές
\bullet Η δέσμη H\ldotp CMBX είναι αρμονική, λόγω MB = MC και HX\parallel BC και τεμνόμενη από την ευθεία EF μας δίνει την αρμονική σημειοσειρά X,\ F,\ S,\ E.

Η ευθεία ZH τώρα, που συνδέει το σημείο H, επαφής της εφαπτομένης του κύκλου (T) από το σημείο X με το σημείο S, ως το αρμονικό συζυγές του X ως προς τα σημεία F,\ E, ταυτίζεται με την Πολική ευθεία του σημείου X ως προς τον κύκλο (T) και άρα ισχύει XT\perp ZH\ \ \ ,(1)

Από (1) και ZH\parallel TO\Rightarrow XT\perp TO και το ζητούμενο έχει αποδειχθεί.

Κώστας Βήττας.

ΥΓ. Για την ολοκλήρωση της απόδειξης \angle XDO = 90^{o}, σύμφωνα με το σκεπτικό της λύσης του padlock, στην παραπομπή που δίνει ο Μπάμπης πιο πάνω, η εγγραψιμότητα του τετραπλέυρου TKOD προκύπτει άμεσα από τα εγγράψιμα τετράπλευρα DTEQ,\ OKEQ, με Q το μέσον της πλευράς AC, από όπου έχουμε (AK)(AO) = (AE)(AQ) = (AT)(AD).


Άβαταρ μέλους
vittasko
Επιμελητής
Δημοσιεύσεις: 2230
Εγγραφή: Πέμ Ιαν 08, 2009 8:46 am
Τοποθεσία: Μαρούσι - Αθήνα.
Επικοινωνία:

Re: Απρόσμενη καθετότητα

#8

Μη αναγνωσμένη δημοσίευση από vittasko » Τετ Δεκ 11, 2019 12:46 am

ΦΩΤΙΑΔΗΣ ΠΡΟΔΡΟΜΟΣ έγραψε:
Τρί Δεκ 10, 2019 6:14 pm
Γενίκευση:

Έστω τρίγωνο ABC και AD ύψος.Έστω P σημείο του ύψους και E\equiv BP\cap AC,F\equiv CP\cap AB.Αν η παράλληλη από το P στην BC τέμνει την EF στο X και Y το ισογώνιο συζυγές σημείο του X ως προς το τρίγωνο ABC να δειχθεί ότι \angle XDY =90^{\circ}
Ωραία γενίκευση για όποιον ενδιαφέρεται.

Απ' όλα έχει ο μπαξές. Αρμονικότητα, Ισογωνιότητα, Deasarques και Διπλούς λόγους.

Κώστας Βήττας.

ΥΓ. Την αφήνω για λίγες μέρες και θα επανέλθω αν δεν απαντηθεί. Ίσως να αποδεικνύεται και με πιο στοιχειώδη μέσα.


min##
Δημοσιεύσεις: 342
Εγγραφή: Τρί Απρ 18, 2017 3:40 pm

Re: Απρόσμενη καθετότητα

#9

Μη αναγνωσμένη δημοσίευση από min## » Τετ Δεκ 11, 2019 3:06 pm

Καλησπέρα και από μένα.
Μια λύση που προέκυψε στην προσπάθεια να αποφύγω τα κινούμενα σημεία :
prosd.png
prosd.png (36.55 KiB) Προβλήθηκε 1013 φορές
Ας είναι X'\equiv AX \cap BC,S\equiv EF \cap BC,A'\equiv AO \cap (ABC),Q\equiv AX \cap (ABC)
Τότε X(A,P,E,D)=-1 από το πλήρες τετράπλευρο \Rightarrow το X' είναι το μέσο του DS.
Είναι X'Q\cdot X'A=X'B\cdot X'C=(Newton)X'D^2.Συνεπώς το Q είναι η τομή των (ABC),(AD),
από όπου προκύπτει εύκολα πως A',D,Q συνευθειακά.Αν ορίσω L\equiv A'D\cap PX,το X θα είναι το ορθόκεντρο του ALD,δηλαδή DX,AL κάθετες,δηλαδή αρκεί AL//DY.
Έστω τώρα H_{A}\equiv AD \cap (ABC).Από αντίστροφο Θαλή,είναι αρκετό να δειχτεί πως \displaystyle \frac{H_{A}D}{DP}=\dfrac{A'Y}{YA}.
Αυτό προκύπτει ως εξής:
Ισχύει B(P,H_{A},D,H)=B(Y,\infty_{AO},A,O)\Rightarrow \dfrac{PD}{H_{A}D}\dfrac{HH_{A}}{HP}= \dfrac{AY}{YO} λόγω συμμετρίας ως προς την διχοτόμο της B\angle.
Εγώ θέλω \dfrac{AY}{YA'}=\dfrac{PD}{DH_{A}}\Rightarrow \dfrac{AY}{YA'}=\dfrac{AY}{YO}\dfrac{HP}{HH_{A}}\Rightarrow \dfrac{YO}{YA'}=\dfrac{HP}{HH_{A}} το οποίο έπεται από την ισότητα διπλών λόγων (λόγω συμμετρίας όπως και πριν) B(P,H_{A},H,\infty_{AD})=B(Y,\infty_{AO},O,A')...


Άβαταρ μέλους
vittasko
Επιμελητής
Δημοσιεύσεις: 2230
Εγγραφή: Πέμ Ιαν 08, 2009 8:46 am
Τοποθεσία: Μαρούσι - Αθήνα.
Επικοινωνία:

Re: Απρόσμενη καθετότητα

#10

Μη αναγνωσμένη δημοσίευση από vittasko » Τετ Ιαν 15, 2020 12:19 pm

Ας δούμε μία άλλη απόδειξη της γενίκευσης, βασισμένη στο αποτέλεσμα της καθετότητας για την περίπτωση του ορθόκεντρου H του \vartriangle ABC ( αρχικό πρόβλημα ).

\bullet Έστω K\in AH τυχόν σημείο και L\in AO το ισογώνιο σημείο του K ως προς το \vartriangle ABC και έστω τα σημεία M\equiv AC\cap BK και N\equiv AB\cap CK.

Από το πλήρες τετράπλευρο AEHFBC έχουμε ότι η ευθεία EF περνάει από το σημείο έστω S της ευθείας BC, ως το αρμονικό συζυγές του D, ως προς τα σημεία B,\ C.

Από το σημείο S περνάει επίσης και η ευθεία MN, ως διαγώνια στο πλήρες τετράπλευρο AMKNBC, αφού D\equiv BC\cap AH\equiv BC\cap AK.

Επειδή τα σημεία S\equiv XE\cap YM και B\equiv HE\cap KM και \infty\equiv XH\cap YK είναι συνευθειακά, σύμφωνα με το Θεώρημα Desarques, έχουμε ότι τα τρίγωνα \vartriangle HEX,\ \vartriangle KMY είναι προοπτικά και άρα, η ευθεία XY περνάει από το σημείο A\equiv HK\cap EM, όπου X\in EF ώστε XH\parallel BC και Y\in MN ώστε YK\parallel BC.
f 181_t 65483a.PNG
Απόδειξη της γενίκευσης του Φωτιάδη Πρόδρομου.
f 181_t 65483a.PNG (34.02 KiB) Προβλήθηκε 852 φορές
\bullet Οι δέσμες S.AXYD,\ D.AXYS με AX\equiv DS και συνευθειακά τα σημεία A\equiv SA\cap DA και X\equiv AX\cap DX και Y\equiv AY\cap DY έχουν ίσους Διπλούς λόγους και άρα, ισχύει \boxed{(S.AXYD) = (D.AXYS)}\ \ \ ,(1)

Αλλά, (S.AXYD) = (A,X,Y,Z) = (A,H,K,D) = (B.AHKD)\Rightarrow \boxed{(S.AXYD) = (B.AHKD)}\ \ \ ,(2) όπου Z\equiv BC\cap AY.

Οι δέσμες B.AHKD,\ B.TOLA, με T\equiv BC\cap AO, έχουν ίσες τις γωνίες που σχηματίζουν οι ομόλογες ακτίνες τους ( λόγω της ισογωνιότητας των σημείων H,\ O και K,\ L ) και άρα έχουν ίσους Διπλούς λόγους και επομένως ισχύει \boxed{(B.AHKD) = (B.TOLA)}\ \ \ ,(3)

Από (1),\ (2),\ (3) και \boxed{(B.TOLA) = (D.TOLA)}\Rightarrow \boxed{(D.AXYS) = (D.TOLA)}\ \ \ ,(4)

Από (4) και επειδή ισχύει DA\perp DT και DX\perp DO και DS\perp DA, συμπεραίνεται ότι ισχύει και \boxed{DY\perp DL} και το ζητούμενο έχει αποδειχθεί.

Κώστας Βήττας.


Απάντηση

Επιστροφή σε “Γεωμετρία - Επίπεδο Αρχιμήδη (Seniors)”

Μέλη σε σύνδεση

Μέλη σε αυτήν τη Δ. Συζήτηση: Δεν υπάρχουν εγγεγραμμένα μέλη και 0 επισκέπτες